Neurological Disorders

Ace your homework & exams now with Quizwiz!

The nurse is admitting the client for rule-out encephalitis. Which interventions should the nurse assess to support the diagnosis of encephalitis? Select all that apply. 1. Determine if the client has recently received any immunizations. 2. Ask the client if he or she has had a cold in the last week. 3. Check to see if the client has active herpes simplex 1. 4. Find out if the client has traveled to the Great Lakes region. 5. Assess for exposure to soil with fungal spores.

1. A complication of immunizations for measles, mumps, and rubella can be encephalitis. 2. Upper respiratory tract illnesses can be a precursor to encephalitis. 3. The herpes simplex virus, specifically type 1, can lead to encephalitis. Test-taking Hint: Encephalitis is in- flammation of the brain caused by either a hypersensitivity reaction or a postinfectious state in which a virus reproduces in the brain. Encephalitis can be a life-threatening disease process. History is vital in the diagnosis.

The nurse is preparing a client diagnosed with rule-out meningitis for a lumbar puncture. Which interventions should the nurse implement? Select all that apply. 1. Obtain an informed consent from the client or significant other. 2. Have the client empty the bladder prior to the procedure. 3. Place the client in a side-lying position with the back arched. 4. Instruct the client to breathe rapidly and deeply during the procedure. 5. Explain to the client what to expect during the procedure.

1. A lumbar puncture is an invasive procedure; therefore, an informed consent is required. 2. This could be offered for client comfort during the procedure. 3. This position increases the space between the vertebrae, which allows the HCP easier entry into the spinal column. 5. The nurse should always explain to the client what is happening prior to and during a procedure. Test-taking Hint: This is an alternative- type question, which requires the test taker to select more than one answer option.

The nurse is caring for a client diagnosed with meningitis. Which collaborative intervention should be included in the plan of care? 1. Administer antibiotics. 2. Obtain a sputum culture. 3. Monitor the pulse oximeter. 4. Assess intake and output.

1. A nurse administering antibiotics is a collaborative intervention because the HCP must write an order for the intervention; nurses cannot prescribe medications unless they have additional education and licensure and are nurse practitioners with prescriptive authority. Test-taking Hint: The test taker must note adjectives and understand that a collaborative nursing intervention is dependent on another member of the health-care team; an independent nursing intervention does not require collaboration.

The client with end-stage ALS requires a gastrostomy tube feeding. Which finding would require the nurse to hold a bolus tube feeding? 1. A residual of 125 mL. 2. The abdomen is soft. 3. Three episodes of diarrhea. 4. The potassium level is 3.4 mEq/L.

1. A residual (aspirated gastric contents) of greater than 50 to 100 mL indicates that the tube feeding is not being digested and that the feeding should be held. Test-taking Hint: Knowing normal assessment data would lead the test taker to eliminate option "2" as a possible correct answer. Diarrhea and hypokalemia would not cause the client to not receive a feeding. Even if the test taker did not know what "residual" means, this would be the best option.

The significant other of a client diagnosed with a brain tumor asks the nurse for help identifying resources. Which would be the most appropriate referral for the nurse to make? 1. Social worker. 2. Chaplain. 3. Health-care provider. 4. Occupational therapist.

1. A social worker is qualified to assist the client with referrals to any agency or personnel that is needed. Test-taking Hint: The test taker must decide what each discipline has to offer the client; a social worker has the broadest range of referral capabilities.

Which client would the nurse identify as being most at risk for experiencing a CVA? 1. A 55-year-old African American male. 2. An 84-year-old Japanese female. 3. A 67-year-old Caucasian male. 4. A 39-year-old pregnant female.

1. African Americans have twice the rate of CVAs as Caucasians and men have a higher incidence than women; African Americans suffer more extensive damage from a CVA than do people of other cultural groups. Reasons for other answers: 2. Females are less likely to have a CVA than males, but advanced age does increase the risk for CVA. The Oriental population has a lower risk, possibly as a result of their relatively high intake of omega-3 fatty acids, antioxidants found in fish. 3. Caucasians have a lower risk of CVA than do African Americans, Hispanics, and Native Pacific Islanders. 4. Pregnancy is a minimal risk for having a CVA. Test-taking Hint: Note the age of the client if this information is given, but take this information in context with the additional information provided in the answer options. The 84-year-old may appear to be the best answer, but not if the client is a female and Oriental, which rules out this option for the client most at risk.

Which is an expected outcome for a client diagnosed with encephalitis? 1. The client will regain as much neurological function as possible. 2. The client will have no short-term memory loss. 3. The client will have improved renal function. 4. The client will apply hydrocortisone cream daily.

1. Clients diagnosed with encephalitis have neurological deficits while the inflammation is present. The thera- peutic plan is to treat the disease process, decrease the edema, and re- turn the client to an optimal level of wellness. Test-taking Hint: The test taker should look at the option that reflects the body system that is involved with the disease. Refer to medical terminology; encephalon means "the brain."

The nurse is developing a plan of care for a client diagnosed with West Nile virus. Which intervention should the nurse include in this plan? 1. Monitor the client's respirations frequently. 2. Refer to dermatologist for treatment of maculopapular rash. 3. Treat hypothermia by using ice packs under the client's arms. 4. Teach the client to report any swollen lymph glands.

1. Clients with West Nile virus should be continuously assessed for alteration in gas exchanges or patterns. Test-taking Hint: The test taker needs to read words carefully. Prefixes such as "hypo-" and "hyper-" are important in determining if an option is correct. Even if the test taker did not know if the client is hypothermic or hyperthermic, "hypo-" means "less than normal" so hypothermia would not be treated with ice packs. A client with West Nile virus usually has a fever that should be reduced. Thus, a treatment for hypothermia is not needed.

Which intervention should the nurse implement when caring for the client diagnosed with encephalitis? Select all that apply. 1. Turn the client every two (2) hours. 2. Encourage the client to increase fluids. 3. Keep the client in the supine position. 4. Assess for deep vein thrombosis. 5. Assess for any alterations in elimination.

1. Clients with encephalitis should be treated for the disease process and also to prevent complications of immobility. Turning the client will prevent skin breakdown. 2. Increasing fluids helps prevent urinary tract infections and mobilize secretions in the lungs. 4. Immobility causes clients to be at risk for deep vein thrombosis. Therefore, clients with encephalitis should be assessed for deep vein thrombosis. 5. Immobility causes the gastrointestinal tract to slow, resulting in constipa- tion. Clients can have difficulty emptying their bladders, which can cause retention and urinary tract infections and stones. Assessing these systems can identify problems early. Test-taking Hint: Each option should be read carefully. If the test taker does not read each one carefully, the test taker could miss important words, such as "supine" in option "3," resulting in an incorrect answer.

The client is in the terminal stage of ALS. Which intervention should the nurse implement? 1. Perform passive ROM every two (2) hours. 2. Maintain a negative nitrogen balance. 3. Encourage a low-protein, soft-mechanical diet. 4. Turn the client and have him cough and deep breathe every shift.

1. Contractures can develop within a week because extensor muscles are weaker than flexor muscles. If the client cannot perform ROM exercises, then the nurse must do it for him—passive ROM. Test-taking Hint: "Terminal stage" is the key term in the stem that should cause the test taker to look for an option addressing immobility issues—option "1." An intervention implemented only once in every shift should be eliminated as a possible answer when addressing immobility issues.

The client is diagnosed with ALS. Which client problem would be most appropriate for this client? 1. Disuse syndrome. 2. Altered body image. 3. Fluid and electrolyte imbalance. 4. Alteration in pain.

1. Disuse syndrome is associated with complications of bedrest. Clients with ALS cannot move and reposition themselves, and they frequently have altered nutritional and hydration status. Test-taking Hint: The test taker would have to be knowledgeable about ALS to answer this question. This disease is chronic and debilitating over time and leads to wasting of the muscles.

The occupational health nurse is concerned about preventing occupation-related acquired seizures. Which intervention should the nurse implement? 1. Ensure that helmets are worn in appropriate areas. 2. Implement daily exercise programs for the staff. 3. Provide healthy foods in the cafeteria. 4. Encourage employees to wear safety glasses.

1. Head injury is one of the main reasons for epilepsy that can be prevented through occupational safety precautions and highway safety programs. Reasons for other answers: 2. Sedentary lifestyle is not a cause of epilepsy. 3. Dietary concerns are not a cause of epilepsy. 4. Safety glasses will help prevent eye injuries, but such injuries are not a cause of epilepsy. Test-taking Hint: The nurse must be aware of risk factors that cause diseases. If the test taker does not know the correct answer, thinking about which body system the question is asking about may help rule out or rule in some of the answer options. Only options "1" and "4" have anything to do with the head, and only helmets on the head are connected with the neurological system.

The charge nurse is making assignments. Which client should be assigned to the new graduate nurse? 1. The client diagnosed with aseptic meningitis who is complaining of a headache and the light bothering his eyes. 2. The client diagnosed with Parkinson's disease who fell during the night and is complaining of difficulty walking. 3. The client diagnosed with a cerebrovascular accident whose vitals signs are P 60, R 14, and BP 198/68. 4. The client diagnosed with a brain tumor who has a new complaint of seeing spots before the eyes.

1. Headache and photophobia are expected clinical manifestations of meningitis. The new graduate could care for this client. Test-taking Hint: The test taker should determine if the clinical manifestations are expected as part of the disease process. If they are, a new graduate can care for the client; if they are not expected occurrences, a more experienced nurse should care for the client.

The nurse is assessing the client diagnosed with meningococcal meningitis. Which assessment data would warrant notifying the HCP? 1. Purpuric lesions on the face. 2. Complaints of light hurting the eyes. 3. Dull, aching, frontal headache. 4. Not remembering the day of the week.

1. In clients with meningococcal meningitis, purpuric lesions over the face and extremity are the signs of a fulminating infection that can lead to death within a few hours. Test-taking Hint: The stem is asking the nurse to identify which assessment data are abnormal for the disease process and require an immediate medical intervention to prevent the client from experiencing a complication or possible death.

The public health department nurse is preparing a lecture on prevention of West Nile virus. Which information should the nurse include? 1. Change water daily in pet dishes and birdbaths. 2. Wear thick, dark clothing when outside to avoid bites. 3. Apply insect repellent over face and arms only. 4. Explain that mosquitoes are more prevalent in the morning.

1. Mosquitoes breed in standing water, even pet dishes and birdbaths. All areas that collect water should be emptied, removed, covered, or turned over. Rain gutters should be cleaned. Test-taking Hint: Words such as "only" (option "3") should clue the test taker to eliminate that option. Rarely are these absolute terms correct.

The chief executive officer (CEO) of a large manufacturing plant presents to the occupational health clinic with chronic rhinitis and requesting medication. On inspection, the nurse notices holes in the septum that separates the nasal passages. The nurse also notes dilated pupils and tachycardia. The facility has a "No Drug" policy. Which intervention should the nurse implement? 1. Prepare to complete a drug screen urine test. 2. Discuss the client's use of illegal drugs. 3. Notify the client's supervisor about the situation. 4. Give the client an antihistamine and say nothing.

1. No employee of a facility is above certain rules. In a company with a "No Drugs" policy, this includes the CEO. This client is exhibiting symptoms of cocaine abuse. Test-taking Hint: The title of the client—CEO—eliminates option "3." The nurse has noted a potential illegal situation.

The nurse enters the room as the client is beginning to have a tonic-clonic seizure. What action should the nurse implement first? 1. Note the first thing the client does in the seizure. 2. Assess the size of the client's pupils. 3. Determine if the client is incontinent of urine or stool. 4. Provide the client with privacy during the seizure.

1. Noticing the first thing the client does during a seizure provides information and clues as to the location of the seizure in the brain. It is important to document whether the beginning of the seizure was observed. Reasons for other answers: 2. Assessment is important, but during the seizure the nurse should not attempt to restrain the head to assess the eyes; muscle contractions are strong, and restraining the client could cause injury. 3. This should be done, but it is not the first intervention when walking into a room where the client is beginning to have a seizure. 4. The client should be protected from onlookers, but the nurse should always address the client first. Test-taking Hint: This is a prioritizing question that asks the test taker which intervention to implement first. All four interventions would be appropriate, but only one should be implemented first. If the test taker cannot decide between two choices, always select the one that directly affects the client or the condition; privacy is important, but helping determine the origin of the seizure is priority.

The nurse in the neurointensive care unit is caring for a client with a new C6 SCI who is breathing independently. Which nursing interventions should be implemented? Select all that apply. 1. Monitor the pulse oximetry reading. 2. Provide pureed foods six (6) times a day. 3. Encourage coughing and deep breathing. 4. Assess for autonomic dysreflexia. 5. Administer intravenous corticosteroids.

1. Oxygen is administered initially to prevent hypoxemia, which can worsen the spinal cord injury; therefore, the nurse should determine how much oxygen is reaching the periphery. 3. Breathing exercises are supervised by the nurse to increase the strength and endurance of inspiratory muscles, especially those of the diaphragm. 5. Corticosteroids are administered to decrease inflammation, which will decrease edema, and help prevent edema from ascending up the spinal cord, causing breathing difficulties. Reasons for other answers: 2. A C6 injury would not affect the client's ability to chew and swallow, so pureed food is not necessary. 4. Autonomic dysreflexia occurs during the rehabilitation phase, not the acute phase. Test-taking Hint: The test taker must notice where the client is receiving care, which may be instrumental in being able to rule out incorrect answer options and help in identifying the correct answer. Remember Maslow's hierarchy of needs— oxygen and breathing are priority nursing interventions.

The client diagnosed with a right-sided cerebrovascular accident is admitted to the rehabilitation unit. Which interventions should be included in the nursing care plan? Select all that apply. 1. Position the client to prevent shoulder adduction. 2. Turn and reposition the client every shift. 3. Encourage the client to move the affected side. 4. Perform quadriceps exercises three (3) times a day. 5. Instruct the client to hold the fingers in a fist.

1. Placing a small pillow under the shoulder will prevent the shoulder from adducting toward the chest and developing a contracture. 3. The client should not ignore the paralyzed side, and the nurse must encourage the client to move it as much as possible; a written schedule may assist the client in exercising. Reasons for other answers: 2. The client should be repositioned at least every two (2) hours to prevent contractures, pneumonia, skin breakdown, and other complications of immobility. 4. These exercises are recommended, but they must be done at least five (5) times a day for 10 minutes to help strengthen the muscles for walking. 5. The fingers are positioned so that they are barely flexed to help prevent contracture of the hand. Test-taking Hint: Be sure to look at the intervals of time for any intervention; note that "every shift" and "three (3) times a day" are not appropriate time intervals for this client. Because this is a "select all that apply" question, the test taker must read each answer option and decide if it is correct; one will not eliminate another.

The client diagnosed with a gunshot wound to the head assumes decorticate posturing when the nurse applies painful stimuli. Which assessment data obtained three (3) hours later would indicate the client is improving? 1. Purposeless movement in response to painful stimuli. 2. Flaccid paralysis in all four extremities. 3. Decerebrate posturing when painful stimuli are applied. 4. Pupils that are 6 mm in size and nonreactive on painful stimuli.

1. Purposeless movement indicates that the client's cerebral edema is decreasing. The best motor response is purposeful movement, but purposeless movement indicates an improvement over decorticate movement, which, in turn, is an improvement over decerebrate movement or flaccidity. Reasons for other answers: 2. Flaccidity would indicate a worsening of the client's condition. 3. Decerebrate posturing would indicate a worsening of the client's condition. 4. The eyes respond to light, not painful stimuli, but a 6-mm nonreactive pupil indicates severe neurological deficit. Test-taking Hint: The test taker must have strong assessment skills and know what specific signs/symptoms signify for each of the body systems—in this case, the significance of different stages of posturing/movement in assessing neurological status.

In assessing a client with a T12 SCI, which clinical manifestations would the nurse expect to find to support the diagnosis of spinal shock? 1. No reflex activity below the waist. 2. Inability to move upper extremities. 3. Complaints of a pounding headache. 4. Hypotension and bradycardia.

1. Spinal shock associated with SCI represents a sudden depression of reflex activity below the level of the injury. T12 is just above the waist; therefore, no reflex activity below the waist would be expected. Reasons for other answers: 2. Assessment of the movement of the upper extremities would be more appropriate with a higher level injury; an injury in the cervical area might cause an inability to move the upper extremities. 3. Complaints of a pounding headache are not typical of a T12 spinal injury. 4. Hypotension (low blood pressure) and tachycardia (rapid heart rate) are signs of hypovolemic or septic shock, but these do not occur in spinal shock. Test-taking Hint: If the test taker does not have any idea what the answer is, an attempt to relate the anatomical position of keywords in the question stem to words in the answer options is appropri- ate. In this case, T12, mentioned in the stem, is around the waist, so answer options involving the anatomy above that level (e.g., the upper extremities) can be eliminated.

The nurse caring for a client who has been abusing amphetamines writes a problem of "cardiovascular compromise." Which nursing interventions should be implemented? 1. Monitor the telemetry and vital signs every four (4) hours. 2. Encourage the client to verbalize the reason for using drugs. 3. Provide a quiet, calm atmosphere for the client to rest. 4. Place the client on bedrest and a low-sodium diet.

1. Telemetry and vital signs would be done to monitor cardiovascular compromise. Amphetamine use causes tachycardia, vasoconstriction, hypertension, and arrhythmias. Test-taking Hint: The correct answer must address the problem of cardiovascular compromise, which eliminates options "2" and "3."

The nurse is working with clients in a substance abuse clinic. Client A tells the nurse that another client, Client B, has "started using again." Which action should the nurse implement? 1. Tell Client A the nurse cannot discuss Client B with him. 2. Find out how Client A got this information. 3. Inform the HCP that Client B is using again. 4. Get in touch with Client B and have the client come to the clinic.

1. The Health Insurance Portability and Accountability Act (HIPAA) requires that a health-care professional not di- vulge information about one person to an unauthorized person. Test-taking Hint: Nurses are required to practice within the laws of the state and within federal laws. HIPAA is a federal law and applies to all health-care profes- sionals in the United States. Legally the nurse cannot use the information provided by Client A, but morally the nurse might try to identify behavior in Client B that would warrant the nurse's intervention.

The client diagnosed with septic meningitis is admitted to the medical floor at noon. Which health-care provider's order would have the highest priority? 1. Administer an intravenous antibiotic. 2. Obtain the client's lunch tray. 3. Provide a quiet, calm, and dark room. 4. Weigh the client in hospital attire.

1. The antibiotic has the highest priority because failure to treat a bacterial infection can result in shock, systemic sepsis, and death. Test-taking Hint: The nurse must know how to prioritize care. Which intervention has the potential to avoid a complication related to the disease process? Remember the word "priority."

A client diagnosed with a subarachnoid hemorrhage has undergone a craniotomy for repair of a ruptured aneurysm. Which intervention will the intensive care nurse implement? 1. Administer a stool softener b.i.d. 2. Encourage the client to cough hourly. 3. Monitor neurological status every shift. 4. Maintain the dopamine drip to keep BP at 160/90.

1. The client is at risk for increased intracranial pressure whenever performing the Valsalva maneuver, which will occur when straining during defecation. Therefore, stool softeners would be appropriate. Reasons for other answers: 2. Coughing increases intracranial pressure and is discouraged for any client who has had a craniotomy. The client is encouraged to turn and breathe deeply, but not to cough. 3. Monitoring the neurological status is appropriate for this client, but it should be done much more frequently than every shift. 4. Dopamine is used to increase blood pres- sure or to maintain renal perfusion, and a BP of 160/90 is too high for this client. Test-taking Hint: The test taker should always notice if an answer option has a time frame—every shift, every four (4) hours, or daily. Whether or not the time frame is correct may lead the test taker to the correct answer.

The client is being discharged following a transsphenoidal hypophysectomy. Which discharge instructions should the nurse teach the client? Select all that apply. 1. Sleep with the head of the bed elevated. 2. Keep a humidifier in the room. 3. Use caution when performing oral care. 4. Stay on a full liquid diet until seen by the HCP. 5. Notify the HCP if developing a cold or fever.

1. The client should sleep with the head of the bed elevated to promote drainage of the cerebrospinal fluid. 2. Humidified air will prevent drying of the nasal passages. 3. Because the incision for this surgery is just above the gumline, the client should not brush the front teeth. Oral care should be performed using a sponge until the incision has healed. 5. The HCP should be notified if the client develops an infection of any kind. A cold with sinus involvement and sneezing places the client at risk for opening the incision and developing a brain infection. Test-taking Hint: The test taker could choose option "5" because this is a standard instruction for any surgery. The test taker should look for more than one correct answer in an alternative-type question.

The client is diagnosed with a closed head injury and is in a coma. The nurse writes the client problem as "high risk for immobility complications." Which intervention would be included in the plan of care? 1. Position the client with the head of the bed elevated at intervals. 2. Perform active range-of-motion exercises every four (4) hours. 3. Turn the client every shift and massage bony prominences. 4. Explain all procedures to the client before performing them.

1. The head of the client's bed should be elevated to help the lungs expand and prevent stasis of secretions that could lead to pneumonia, a complication of immobility. Reasons for other answers: 2. Active range-of-motion exercises require that the client participate in the activity. This is not possible because the client is in a coma. 3. The client is at risk for pressure ulcers and should be turned more frequently than every shift, and research now shows that massaging bony prominences can increase the risk for tissue breakdown. 4. The nurse should always talk to the client, even if he or she is in a coma, but this will not address the problem of immobility. Test-taking Hint: Whenever a client problem is written, interventions must address the specific problem, not the disease. Positioning the client addresses the possibility of immobility complications, whereas talking to a comatose client addresses communication deficit and psychosocial needs, not immobility issues.

The nurse and the unlicensed assistive personnel (UAP) are caring for clients on a medical-surgical unit. Which task should not be assigned to the UAP? 1. Feed the 69-year-old client diagnosed with Parkinson's disease who is having difficulty swallowing. 2. Turn and position the 89-year-old client diagnosed with a pressure ulcer secondary to Parkinson's disease. 3. Assist the 54-year-old client diagnosed with Parkinson's disease with toilet-training activities. 4. Obtain vital signs on a 72-year-old client diagnosed with pneumonia secondary to Parkinson's disease.

1. The nurse should not delegate feeding a client who is at risk for complications during feeding. This requires judgment that the UAP is not expected to possess. Test-taking Hint: When reading the answer options in a question in which the nurse is delegating to an unlicensed assistive personnel, read the stem carefully. Is the question asking what to delegate or what not to delegate? Anything requiring professional judgment should not be delegated.

The client diagnosed with atrial fibrillation has experienced a transient ischemic attack (TIA). Which medication would the nurse anticipate being ordered for the client on discharge? 1. An oral anticoagulant medication. 2. A beta blocker medication. 3. An anti-hyperuricemic medication. 4. A thrombolytic medication.

1. The nurse would anticipate an oral anticoagulant, warfarin (Coumadin), to be prescribed to help prevent thrombi formation in the atria secondary to atrial fibrillation. The thrombi can become embolic and may cause a TIA or CVA (stroke). Reasons for other answers: 2. Beta blockers slow the heart rate and decrease blood pressure but would not be an anticipated medication to help prevent a TIA secondary to atrial fibrillation. 3. An anti-hyperuricemic medication is administered for a client experiencing gout and decreases the formation of tophi. 4. A thrombolytic medication is administered to dissolve a clot, and it may be ordered during the initial presentation for a client with a CVA, but not on discharge. Test-taking Hint: In the stem of this question, there are two disease processes mentioned—atrial fibrillation and TIA. The reader must determine how one process affects the other before answering the question. In this question, the test taker must know atrial fibrillation predisposes the client to the formation of thrombi, and therefore the nurse should anticipate the health-care provider ordering a medication to prevent clot formation, an anticoagulant.

The client diagnosed with delirium tremens when trying to quit drinking cold turkey is admitted to the medical unit. Which medications would the nurse anticipate administering? 1. Thiamine (vitamin B6) and librium, a benzodiazepine. 2. Dilantin, an anticonvulsant, and Feosol, an iron preparation. 3. Methadone, a synthetic narcotic, and Depakote, a mood stabilizer. 4. Mannitol, an osmotic diuretic, and Ritalin, a stimulant.

1. Thiamine is given in high doses to decrease the rebound effect on the nervous system as it adjusts to the absence of alcohol, and a benzodi- azepine is given in high doses and titrated down over several days for the tranquilizing effect to prevent delirium tremens. Test-taking Hint: Option "3" could be eliminated if the test taker knew the treatment for heroin withdrawal, and option "4" could be reasoned out because a stimulant would produce an undesired effect.

The nurse on the rehabilitation unit is caring for the following clients. Which client should the nurse assess first after receiving the change-of-shift report? 1. The client with a C6 SCI who is complaining of dyspnea and has crackles in the lungs. 2. The client with an L4 SCI who is crying and very upset about being discharged home. 3. The client with an L2 SCI who is complaining of a headache and feeling very hot. 4. The client with a T4 SCI who is unable to move the lower extremities.

1. This client has signs/symptoms of a respiratory complication and should be assessed first. Reasons for other answers: 2. This is a psychosocial need and should be addressed, but it does not have priority over a physiological problem. 3. A client with a lower SCI would not be at risk for autonomic dysreflexia; therefore, a complaint of headache and feeling hot would not be priority over an airway problem. 4. The client with a T4 SCI would not be expected to move the lower extremities. Test-taking Hint: The nurse should assess the client who is at risk for dying or having some type of complication that requires intervention. Remember Maslow's hierarchy of needs, in which physiological problems are always priority and airway is the top physiological problem.

The client is prescribed phenytoin (Dilantin), an anticonvulsant, for a seizure disorder. Which statement indicates the client understands the discharge teaching concerning this medication? 1. "I will brush my teeth after every meal." 2. "I will check my Dilantin level daily." 3. "My urine will turn orange while on Dilantin." 4. "I won't have any seizures while on this medication."

1. Thorough oral hygiene after each meal, gum massage, daily flossing, and regular dental care are essential to prevent or control gingival hyperplasia, which is a common occurrence in clients taking Dilantin. Test-taking Hint: The test taker should realize that monitoring blood glucose levels using a glucometer is about the only level that is monitored daily; therefore, option "2," which calls for daily monitoring of Dilantin levels, could be eliminated. Remember, there are very few absolutes in the health-care field; therefore, option "4" could be ruled out because "won't have any" is an absolute.

The nurse asks the male client with epilepsy if he has auras with his seizures. The client says, "I don't know what you mean. What are auras?" Which statement by the nurse would be the best response? 1. "Some people have a warning that the seizure is about to start." 2. "Auras occur when you are physically and psychologically exhausted." 3. "You're concerned that you do not have auras before your seizures?" 4. "Auras usually cause you to be sleepy after you have a seizure."

1.An aura is a visual, auditory, or olfactory occurrence that takes place prior to a seizure and warns the client a seizure is about to occur. The aura often allows time for the client to lie down on the floor or find a safe place to have the seizure. Test-taking Hint: If the stem of the question has the client asking a question, then the nurse needs to give factual information, and option "3," a therapeutic response, would not be appropriate. Neither would option "2" or "4" because these options are worded in such a way as to imply incorrect information.

The client has been newly diagnosed with epilepsy. Which discharge instructions should be taught to the client? Select all that apply. 1. Keep a record of seizure activity. 2. Take tub baths only; do not take showers. 3. Avoid over-the-counter medications. 4. Have anticonvulsant medication serum levels checked regularly. 5. Do not drive alone; have someone in the car.

1.Keeping a seizure and medication chart will be helpful when keeping follow-up appointments with the health-care provider and in identifying activities that may trigger a seizure. 3. Over-the-counter medications may contain ingredients that will interact with antiseizure medications or, in some cases, as with use of stimulants, possibly cause a seizure. 4. Most of the anticonvulsant medications have therapeutic serum levels that should be maintained, and regular checks of the serum levels help to ensure the correct level. Test-taking Hint: The test taker must select all interventions that are appropriate for the question. A key word is the adverb "newly."

The 29-year-old client who was employed as a forklift operator sustains a traumatic brain injury secondary to a motor-vehicle accident. The client is being discharged from the rehabilitation unit after three (3) months and has cognitive deficits. Which goal would be most realistic for this client? 1. The client will return to work within six (6) months. 2. The client is able to focus and stay on task for 10 minutes. 3. The client will be able to dress self without assistance. 4. The client will regain bowel and bladder control.

2. "Cognitive" pertains to mental processes of comprehension, judgment, memory, and reasoning. Therefore, an appropriate goal would be for the client to stay on task for 10 minutes. Reasons for other answers: 1. The client is at risk for seizures and does not process information appropriately. Allowing him to return to his occupation as a forklift operator is a safety risk for him and other employees. Vocational training may be required. 3. The client's ability to dress self addresses self-care problems, not a cognitive problem. 4. The client's ability to regain bowel and bladder control does not address cognitive deficits. Test-taking Hint: The test taker must note adjectives closely. The question is asking about "cognitive" deficits; therefore, the correct answer must address cognition.

The nurse is developing a plan of care for a client diagnosed with aseptic meningitis secondary to a brain tumor. Which nursing goal would be most appropriate for the client problem "altered cerebral tissue perfusion"? 1. The client will be able to complete activities of daily living. 2. The client will be protected from injury if seizure activity occurs. 3. The client will be afebrile for 48 hours prior to discharge. 4. The client will have elastic tissue turgor with ready recoil.

2. A client with a problem of altered cerebral tissue perfusion is at risk for seizure activity secondary to focal areas of cortical irritability; therefore, the client should be on seizure precautions. Test-taking Hint: The goal must be related to the problem—in this case, "altered cerebral tissue perfusion."

The client diagnosed with ALS asks the nurse, "I know this disease is going to kill me. What will happen to me in the end?" Which statement by the nurse would be most appropriate? 1. "You are afraid of how you will die?" 2. "Most people with ALS die of respiratory failure." 3. "Don't talk like that. You have to stay positive." 4. "ALS is not a killer. You can live a long life."

2. About 50% of clients die within two (2) to five (5) years from respiratory failure, aspiration pneumonia, or another infectious process. Test-taking Hint: When the client is asking for factual information, the nurse should provide accurate and truthful information. This helps foster a trusting client-nurse relationship. A therapeutic response (option "1") should be used when the client needs to ventilate feelings and is not asking specific questions about the disease process.

Which problem is the highest priority for the client diagnosed with West Nile virus? 1. Alteration in body temperature. 2. Altered tissue perfusion. 3. Fluid volume excess. 4. Altered skin integrity.

2. Altered tissue perfusion would be the highest priority because it could be life threatening. Test-taking Hint: When prioritizing client problems, oxygenation is the highest priority problem according to Maslow, and tissue perfusion is oxygenation.

The client diagnosed with a closed head injury is admitted to the rehabilitation department. Which medication order would the nurse question? 1. A subcutaneous anticoagulant. 2. An intravenous osmotic diuretic. 3. An oral anticonvulsant. 4. An oral proton pump inhibitor.

2. An osmotic diuretic would be ordered in the acute phase to help decrease cerebral edema, but this medication would not be expected to be ordered in a rehabilitation unit. Reasons for other answers: 1. The client in rehabilitation is at risk for the development of deep vein thrombosis; therefore, this is an appropriate medication. 3. Clients with head injuries are at risk for post-traumatic seizures; thus an oral anticonvulsant would be administered for seizure prophylaxis. 4. The client is at risk for a stress ulcer; therefore, an oral proton pump inhibitor would be an appropriate medication. Test-taking Hint: The client is in the reha- bilitation unit and therefore must be stable. The use of any intravenous medication should be questioned under those circum- stances, even if the test taker is not sure why the medication may be considered.

The client has undergone a craniotomy for a brain tumor. Which data indicate a complication of this surgery? 1. The client complains of a headache at "3" to "4" on a 1-to-10 scale. 2. The client has an intake of 1,000 mL and an output of 3,500 mL. 3. The client complains of a raspy sore throat. 4. The client experiences dizziness when trying to get up too quickly.

2. An output much larger than the intake could indicate the development of diabetes insipidus. Pressure on the pituitary gland can result in decreased production of vasopressin, the antidiuretic hormone. Test-taking Hint: The test taker could eliminate options "1" and "3" as expected occurrences following the surgery and not complications. Option "4" can also be expected.

Which diagnostic test is used to confirm the diagnosis of ALS? 1. Electromyogram (EMG). 2. Muscle biopsy. 3. Serum creatine kinase (CK). 4. Pulmonary function test.

2. Biopsy confirms changes consistent with atrophy and loss of muscle fiber, both characteristic of ALS. Test-taking Hint: The test taker must be clear as to what the question is asking. The word "confirm" is the key to answering this question correctly. The test taker would need to know that this disease affects the muscle tissue to correctly identify the answer.

The client is diagnosed with meningococcal meningitis. Which preventive measure would the nurse expect the health-care provider to order for the significant others in the home? 1. The Haemophilus influenzae vaccine. 2. Antimicrobial chemoprophylaxis. 3. A 10-day dose pack of corticosteroids. 4. A gamma globulin injection.

2. Chemoprophylaxis includes administering medication that will prevent infection or eradicate the bacteria and the development of symptoms in people who have been in close proximity to the client. Medications include rifampin (Rifadin), ciprofloxacin (Cipro), and ceftriaxone (Rocephin). Test-taking Hint: The key word in the stem is "preventive." The test taker must pay close attention to the adjectives.

The rehabilitation nurse caring for the client with an L1 SCI is developing the nursing care plan. Which intervention should the nurse implement? 1. Keep oxygen via nasal cannula on at all times. 2. Administer low-dose subcutaneous anticoagulants. 3. Perform active lower extremity ROM exercises. 4. Refer to a speech therapist for ventilator-assisted speech.

2. Deep vein thrombosis (DVT) is a potential complication of immobility, which can occur because the client cannot move the lower extremities as a result of the L1 SCI. Low-dose anticoagulation therapy (Lovenox) helps prevent blood from coagulating, thereby preventing DVTs. Reasons for other answers: 1. Oxygen is administered initially to maintain a high arterial partial pressure of oxygen (PaO2) because hypoxemia can worsen a neurological deficit to the spinal cord initially, but this client is in the rehabilitation department and thus not in the initial stages of the injury. 3. The client is unable to move the lower extremities. The nurse should do passive ROM exercises. 4. A client with a spinal injury at C4 or above would be dependent on a ventilator for breathing, but a client with an L1 SCI would not. Test-taking Hint: The test taker should notice any adjectives such as "rehabilitation," which should clue the test taker into ruling out oxygen, which is for the acute phase. The test taker should also be very selective if choosing an answer with a definitive word such as "all" (option "1").

The client diagnosed with Parkinson's disease (PD) is being admitted with a fever and patchy infiltrates in the lung fields on the chest x-ray. Which clinical manifestations of PD would explain these assessment data? 1. Masklike facies and shuffling gait. 2. Difficulty swallowing and immobility. 3. Pill rolling of fingers and flat affect. 4. Lack of arm swing and bradykinesia.

2. Difficulty swallowing places the client at risk for aspiration. Immobility predisposes the client to pneumonia. Both clinical manifestations place the client at risk for pulmonary complications. Test-taking Hint: The nurse must recognize the clinical manifestations of a disease and the resulting bodily compromise. In this situation, fever and patchy infiltrates on a chest x-ray indicate a pulmonary complication. Options "1," "3," and "4" focus on motor problems and could be ruled out as too similar. Only option "2" includes dissimilar information.

The client is diagnosed with expressive aphasia. Which psychosocial client problem would the nurse include in the plan of care? 1. Potential for injury. 2. Powerlessness. 3. Disturbed thought processes. 4. Sexual dysfunction.

2. Expressive aphasia means that the client cannot communicate thoughts but understands what is being communicated; this leads to frustration, anger, depression, and the inability to verbalize needs, which, in turn, causes the client to have a lack of control and feel powerless. Reasons for other answers: 1. Potential for injury is a physiological problem, not a psychosocial problem. 3. A disturbance in thought processes is a cognitive problem; with expressive aphasia the client's thought processes are intact. 4. Sexual dysfunction can have a psychosocial component or a physical component, but it is not related to expressive aphasia. Test-taking Hint: The test taker should always make sure that the choice selected as the correct answer matches what the question is asking. The stem has the adjective "psychosocial," so the correct answer must address psychosocial needs.

Which statement best describes the scientific rationale for alternating a nonnarcotic antipyretic and a nonsteroidal anti-inflammatory drug (NSAID) every two (2) hours to a female client diagnosed with bacterial meningitis? 1. This regimen helps to decrease the purulent exudate surrounding the meninges. 2. These medications will decrease intracranial pressure and brain metabolism. 3. These medications will increase the client's memory and orientation. 4. This will help prevent a yeast infection secondary to antibiotic therapy.

2. Fever increases cerebral metabolism and intracranial pressure. Therefore, measures are taken to reduce body temperature as soon as possible, and alternating Tylenol and Motrin would be appropriate. Test-taking Hint: The test taker must have a basic knowledge of the disease process and medications that are prescribed to treat a disease. Purulent drainage would require an antibiotic. Therefore, option "1" should be eliminated as a possible answer because the question is asking about NSAIDs and a nonnarcotic antipyretic (Tylenol).

A 20-year-old female client who tried lysergic acid diethylamide (LSD) as a teen tells the nurse that she has bad dreams that make her want to kill herself. Which is the explanation for this occurrence? 1. These occurrences are referred to as "holdover reactions" to the drug. 2. These are flashbacks to a time when the client had a "bad trip." 3. The drug is still in the client's body and causing these reactions. 4. The client is suicidal and should be on one-to-one precautions.

2. Flashback reactions occur after the use of hallucinogens in which the client relives a bad episode that occurred while using the drug. Test-taking Hint: The client is 20 years old and took the drug in her teens; drugs do not stay in the body for extended periods. This eliminates option "3."

The public health nurse is giving a lecture on potential outbreaks of infectious meningitis. Which population is most at risk for an outbreak? 1. Clients recently discharged from the hospital. 2. Residents of a college dormitory. 3. Individuals who visit a third world country. 4. Employees in a high-rise office building.

2. Outbreaks of infectious meningitis are most likely to occur in dense community groups such as college campuses, jails, and military installations. Test-taking Hint: The test taker must remember that the NCLEX-RN tests all areas of nursing, so always notice the type of nurse if this is mentioned in the stem. A public health nurse would not be concerned with third world countries.

The client with ALS is admitted to the medical unit with shortness of breath, dyspnea, and respiratory complications. Which intervention should the nurse implement first? 1. Elevate the head of the bed 30 degrees. 2. Administer oxygen via nasal cannula. 3. Assess the client's lung sounds. 4. Obtain a pulse oximeter reading.

2. Oxygen should be given immediately to help alleviate the difficulty breathing. Remember that oxygenation is priority. Test-taking Hint: The test taker should not automatically select assessment. Make sure that there is not another intervention that will directly help the client, especially if the client is experiencing a life-threatening complication.

The client with a cervical fracture is being discharged in a halo device. Which teaching instruction should the nurse discuss with the client? 1. Discuss how to correctly remove the insertion pins. 2. Instruct the client to report reddened or irritated skin areas. 3. Inform the client that the vest liner cannot be changed. 4. Encourage the client to remain in the recliner as much as possible.

2. Reddened areas, especially under the brace, must be reported to the HCP because pressure ulcers can occur when wearing this appliance for an extended period. Reasons for other answers: 1. The halo device is applied by inserting pins into the skull, and the client cannot remove them; the pins should be checked for signs of infection. 3. . The vest liner should be changed for hygiene reasons, but the halo part is not removed. 4. The client should be encouraged to ambulate to prevent complications of immobility. Test-taking Hint: The test taker would need basic knowledge about a halo device to answer this question easily, but some clues are in the stem. A cervical fracture is in the upper portion of the spine or neck area, and most people understand that a halo is something that surrounds the forehead or higher. So the test taker could get a mental image of a device that must span this area of the body and maintain alignment of the neck. If an HCP attaches pins into the head, then the test taker could assume that they were not to be removed by the client. Redness usually indicates some sort of problem with the skin.

The nurse is caring for a client diagnosed with an epidural hematoma. Which nursing interventions should the nurse implement? Select all that apply. 1. Maintain the head of the bed at 60 degrees of elevation. 2. Administer stool softeners daily. 3. Ensure that pulse oximeter reading is higher than 93%. 4. Perform deep nasal suction every two (2) hours. 5. Administer mild sedatives.

2. Stool softeners are initiated to prevent the Valsalva maneuver, which increases intracranial pressure. 3. Oxygen saturation higher than 93% ensures oxygenation of the brain tissues; decreasing oxygen levels increase cerebral edema. 5. Mild sedatives will reduce the client's agitation; strong narcotics would not be administered because they decrease the client's level of consciousness. Reasons for other answers: 1. The head of the bed should be elevated no more than 30 degrees to help decrease cerebral edema by gravity. 4. Noxious stimuli, such as suctioning, increase intracranial pressure and should be avoided. Test-taking Hint: In "select all that apply" questions, the test taker should look at each answer option as a separate entity. In option "1" the test taker should attempt to get a mental picture of the client's position in the bed. A 60-degree angle is almost upright in the bed. Would any client diagnosed with a head injury be placed this high? The client would be at risk for slumping over because of the inability to control the body position. Nasal suctioning, option "4," which increases intracranial pressure, should also be avoided.

The client is diagnosed with a metastatic brain tumor, and radiation therapy is scheduled. The client asks the nurse, "Why not try chemotherapy first? It has helped my other tumors." The nurse's response is based on which scientific rationale? 1. Chemotherapy is only used as a last resort in caring for clients with brain tumors. 2. The blood-brain barrier prevents medications from reaching the brain. 3. Radiation therapy will have fewer side effects than chemotherapy. 4. Metastatic tumors become resistant to chemotherapy and it becomes useless.

2. The blood-brain barrier is the body's defense mechanism for protecting the brain from chemical effects; in this case, it prevents the chemotherapy from being able to work on the tumor in the brain. Test-taking Hint: The test taker can eliminate option "1" as a possible answer because it states that chemotherapy is used to treat brain tumors, but it does not tell the client why it is not being used. Option "2" is the only one that actually informs the client of a medical reason for not administering chemotherapy for a brain tumor

The client is being admitted to rule out a brain tumor. Which classic triad of symptoms supports a diagnosis of brain tumor? 1. Nervousness, metastasis to the lungs, and seizures. 2. Headache, vomiting, and papilledema. 3. Hypotension, tachycardia, and tachypnea. 4. Abrupt loss of motor function, diarrhea, and changes in taste.

2. The classic triad of symptoms suggesting a brain tumor includes a headache that is dull, unrelenting, and worse in the morning; vomiting unrelated to food intake; and edema of the optic nerve (papilledema), which occurs in 70% to 75% of clients diagnosed with brain tumors. Papilledema causes visual disturbances such as decreased visual acuity and diplopia. Test-taking Hint: The test taker can rule out option "4" because of the symptom of diarrhea, which is a gastrointestinal symp- tom, not a neurological one. Considering the other three possible choices, the symptom of "headache" would make sense for a client with a brain tumor.

The client diagnosed with substance abuse is being discharged from a drug and alcohol rehabilitation facility. Which information should the nurse teach the client? 1. "Do not go anyplace where you can be tempted to use again." 2. "It is important that you attend a 12-step meeting regularly." 3. "Now that you are clean, your family will be willing to see you again." 4. "You should explain to all your coworkers what has happened."

2. The client will require a follow-up program such as 12-step meetings if the client is not to relapse. Test-taking Hint: The test taker must notice descriptive words such as "all" or "do not go anywhere." These words or phrases are absolutes that should cause the test taker to eliminate the options containing them.

The nurse is assessing a client experiencing motor loss as a result of a left-sided cere- brovascular accident (CVA). Which clinical manifestations would the nurse document? 1. Hemiparesis of the client's left arm and apraxia. 2. Paralysis of the right side of the body and ataxia. 3. Homonymous hemianopsia and diplopia. 4. Impulsive behavior and hostility toward family.

2. The most common motor dysfunction of a CVA is paralysis of one side of the body, hemiplegia; in this case with a left-sided CVA, the paralysis would affect the right side. Ataxia is an impaired ability to coordinate movement. Reasons for other answers: 1. A left-sided CVA will result in right-sided motor deficits; hemiparesis is weakness of one half of the body, not just the upper extremity. Apraxia, the inability to perform a previously learned task, is a communica- tion loss, not a motor loss. 3. Homonymous hemianopsia (loss of half of the visual field of each eye) and diplopia (double vision) are visual field deficits that a client with a CVA may experience, but they are not motor losses. 4. Personality disorders occur in clients with a right-sided CVA and are cognitive deficits; hostility is an emotional deficit. Test-taking Hint: Be sure to always notice adjectives describing something. In this case, "left-sided" describes the type of CVA. Also be sure to identify exactly what the question is asking—in this case, about "motor loss," which will help rule out many of the possible answer options.

The unlicensed assistive personnel (UAP) is attempting to put an oral airway in the mouth of a client having a tonic-clonic seizure. Which action should the primary nurse take? 1. Help the UAP to insert the oral airway in the mouth. 2. Tell the UAP to stop trying to insert anything in the mouth. 3. Take no action because the UAP is handling the situation. 4. Notify the charge nurse of the situation immediately.

2. The nurse should tell the UAP to stop trying to insert anything in the mouth of the client experiencing a seizure. Broken teeth and injury tothe lips and tongue may result from trying to place anything in the clenched jaws of a client having a tonic-clonic seizure. Reasons for other answers: 1. Once the seizure has started, no one should attempt to put anything in the client's mouth. 3. The primary nurse is responsible for the action of the UAP and should stop the UAP from doing anything potentially dangerous to the client. No one should attempt to pry open the jaws that are clenched in a spasm to insert anything. 4. The primary nurse must correct the action of the UAP immediately, prior to any injury occurring to the client and before notifying the charge nurse. Test-taking Hint: The nurse is responsible for the actions of the unlicensed assistive personnel and must correct the behavior immediately.

The 34-year-old male client with an SCI is sharing with the nurse that he is worried about finding employment after being discharged from the rehabilitation unit. Which intervention should the nurse implement? 1. Refer the client to the American Spinal Cord Injury Association (ASIA). 2. Refer the client to the state rehabilitation commission. 3. Ask the social worker about applying for disability. 4. Suggest that the client talk with his significant other about this concern.

2. The rehabilitation commission of each state will help evaluate and determine if the client can receive training or education for another occupation after injury. Reasons for other answers: 1. The ASIA is an appropriate referral for liv- ing with this condition, but it does not help find gainful employment after the injury. 3. The client is not asking about disability; he is concerned about employment. Therefore, the nurse needs to make a referral to the appropriate agency. 4. This does not address the client's concern about gainful employment. Test-taking Hint: If the question mentions a specific age for a client, the nurse should consider it when attempting to answer the question. This is a young person who needs to find gainful employment. Remember Erickson's stages of growth and development.

The nurse is conducting a support group for clients diagnosed with Parkinson's disease and their significant others. Which information regarding psychosocial needs should be included in the discussion? 1. The client should discuss feelings about being placed on a ventilator. 2. The client may have rapid mood swings and become easily upset. 3. Pill-rolling tremors will become worse when the medication is wearing off. 4. The client may automatically start to repeat what another person says.

2. These are psychosocial manifestations of PD. These should be discussed in the support meeting. Test-taking Hint: Psychosocial problems should address the client's feelings or interactions with another person.

The resident in a long-term care facility fell during the previous shift and has a lacer- ation in the occipital area that has been closed with Steri-Strips. Which signs/ symptoms would warrant transferring the resident to the emergency department? 1. A 4-cm area of bright red drainage on the dressing. 2. A weak pulse, shallow respirations, and cool pale skin. 3. Pupils that are equal, react to light, and accommodate. 4. Complaints of a headache that resolves with medication.

2. These signs/symptoms—weak pulse, shallow respirations, cool pale skin— indicate increased intracranial pressure from cerebral edema secondary to the fall, and they require immediate attention. Reasons for other answers: 1. The scalp is a very vascular area and a moderate amount of bleeding would be expected. 3. This is a normal pupillary response and would not warrant intervention. 4. A headache that resolves with medication is not an emergency situation, and the nurse would expect the client to have a headache after the fall; a headache not relieved with Tylenol would warrant further investigation. Test-taking Hint: The test taker is looking for an answer option that is not normal for the client's situation. Of the options listed, three would be expected and would not warrant a trip to the emergency department.

The male client diagnosed with a brain tumor is scheduled for a magnetic resonance imaging (MRI) scan in the morning. The client tells the nurse that he is scared. Which response by the nurse indicates an appropriate therapeutic response? 1. "MRIs are loud but there will not be any invasive procedure done." 2. "You're scared. Tell me about what is scaring you." 3. "This is the least thing to be scared about—there will be worse." 4. "I can call the MRI tech to come and talk to you about the scan."

2. This is restating and offering self. Both are therapeutic responses. Test-taking Hint: When the question asks the test taker for a therapeutic response, the test taker should choose the response that directly addresses the client's feelings.

The nurse is caring for the client with encephalitis. Which intervention should the nurse implement first if the client is experiencing a complication? 1. Examine pupil reactions to light. 2. Assess level of consciousness. 3. Observe for seizure activity. 4. Monitor vital signs every shift.

2. This is the most important assess- ment data. A change in level of consciousness is usually the first sign of neurological deterioration. Test-taking Hint: The word "first" asks the test taker to prioritize the interventions. Usually all the options are interventions that the nurse should do, but the question implies that the client may be deteriorating. Level of consciousness is the most sensitive indicator of neurological deficit.

The client is to receive a 100-mL intravenous antibiotic over 30 minutes via an intravenous pump. At what rate should the nurse set the IV pump? ___________

200 mL/hr Test-taking Hint: This is a basic calculation that the nurse should be able to make even without a calculator.

A 78-year-old client is admitted to the emergency department with numbness and weakness of the left arm and slurred speech. Which nursing intervention is priority? 1. Prepare to administer recombinant tissue plasminogen activator (rt-PA). 2. Discuss the precipitating factors that caused the symptoms. 3. Schedule for a STAT computed tomography (CT) scan of the head. 4. Notify the speech pathologist for an emergency consult.

3. A CT scan will determine if the client is having a stroke or has a brain tumor or another neurological disorder. If a CVA is diagnosed, the CT scan can determine if it is a hemorrhagic or ischemic accident and guide treatment. Reasoning for other answers: 1. The drug rt-PA may be administered, but a cerebrovascular accident (CVA) must be verified by diagnostic tests prior to administering it. rt-PA helps dissolve a blood clot, and it may be administered if an ischemic CVA is verified; rt-PA is not given if the client is experiencing a hemorrhagic stroke. 2. Teaching is important to help prevent another CVA, but it is not the priority intervention on admission to the emergency department. Slurred speech indicates problems that may interfere with teaching. 4. The client may be referred for speech deficits and/or swallowing difficulty, but referrals are not priority in the emergency department. Test-Taking Hint: When "priority" is used in the stem, all answer options may be appropriate for the client situation, but only one option is priority. The client must have a documented diagnosis before treatment is started.

The nurse educator is presenting an in-service on seizures. Which disease process is the leading cause of seizures in the elderly? 1. Alzheimer's disease. 2. Parkinson's disease. 3. Cerebral vascular accident (stroke). 4. Brain atrophy due to aging.

3. A CVA (stroke) is the leading cause of seizures in the elderly; increased intracranial pressure associated with the stroke can lead to seizures. Test-taking Hint: All four answer options are associated with the brain, neurological system, and aging. However, options "1," "2," and "4" usually occur over time, with the condition gradually getting worse, and thus can be eliminated as a cause of seizures, which are usually sudden.

The client is diagnosed with ALS. As the disease progresses, which intervention should the nurse implement? 1. Discuss the need to be placed in a long-term care facility. 2. Explain how to care for a sigmoid colostomy. 3. Assist the client to prepare an advance directive. 4. Teach the client how to use a motorized wheelchair.

3. A client with ALS usually dies within five (5) years. Therefore, the nurse should offer the client the opportunity to determine how he/she wants to die. Test-taking Hint: The nurse should always help the client prepare for death in disease processes that are terminal and should discuss advance directives, which include both a durable power of attorney for health care and a living will.

The nurse is assessing the client diagnosed with bacterial meningitis. Which clinical manifestations would support the diagnosis of bacterial meningitis? 1. Positive Babinski's sign and peripheral paresthesia. 2. Negative Chvostek's sign and facial tingling. 3. Positive Kernig's sign and nuchal rigidity. 4. Negative Trousseau's sign and nystagmus.

3. A positive Kernig's sign (client unable to extend leg when lying flat) and nuchal rigidity (stiff neck) are signs of bacterial meningitis, occurring because the meninges surrounding the brain and spinal column are irritated. Test-taking Hint: If two answer options test for the same thing (Trousseau's and Chvostek's signs), then the test taker can rule out these as possible answers because there cannot be two correct answers in the question, unless the question tells the test taker that it is a "select all that apply" question.

The client is admitted to the intensive care department (ICD) experiencing status epilepticus. Which collaborative intervention should the nurse anticipate? 1. Assess the client's neurological status every hour. 2. Monitor the client's heart rhythm via telemetry. 3. Administer an anticonvulsant medication by intravenous push. 4. Prepare to administer a glucocorticosteroid orally.

3. Administering an anticonvulsant medication by intravenous push requires the nurse to have an order or confer with another member of the health-care team. Test-taking Hint: The key word in the stem of this question is the adjective "collaborative." The test taker would eliminate the options "1" and "2" because these do not require collaboration with another member of the health-care team and would eliminate option "4" because it is not used to treat seizures.

The client is diagnosed with a pituitary tumor and is scheduled for a transsphenoidal hypophysectomy. Which preoperative instruction is important for the nurse to teach? 1. There will be a large turban dressing around the skull after surgery. 2. The client will not be able to eat for four (4) or five (5) days postop. 3. The client should not blow the nose for two (2) weeks after surgery. 4. The client will have to lie flat for 24 hours following the surgery.

3. Blowing the nose creates increased intracranial pressure and could result in a cerebrospinal fluid leak. Test-taking Hint: The test taker must know the procedures for specific disease processes to answer this question, but anatomical positioning of the pituitary gland (just above the sinuses) could help to eliminate option "1," which calls for a large turban dressing.

The nurse is caring for clients on a medical-surgical floor. Which client should be assessed first? 1. The 65-year-old client diagnosed with seizures who is complaining of a headache that is a "2" on a 1-to-10 scale. 2. The 24-year-old client diagnosed with a T10 spinal cord injury who cannot move his toes. 3. The 58-year-old client diagnosed with Parkinson's disease who is crying and worried about her facial appearance. 4. The 62-year-old client diagnosed with a cerebrovascular accident who has a resolving left hemiparesis.

3. Body image is a concern for clients diagnosed with PD. This client is the one client who is not experiencing expected sequelae of the disease. Test-taking Hint: At times a psychological problem can have priority. All the physical problems are expected and are not life threatening or life altering.

The client diagnosed with PD is being discharged on carbidopa/levodopa (Sinemet), an antiparkinsonian drug. Which statement is the scientific rationale for combining these medications? 1. There will be fewer side effects with this combination than with carbidopa alone. 2. Dopamine D requires the presence of both of these medications to work. 3. Carbidopa makes more levodopa available to the brain. 4. Carbidopa crosses the blood-brain barrier to treat Parkinson's disease.

3. Carbidopa enhances the effects of levodopa by inhibiting decarboxylase in the periphery, thereby making more levodopa available to the central nervous system. Sinemet is the most effective treatment for PD. Test-taking Hint: The nurse must be knowledgeable of the rationale for administering a medication for a specific disease.

The client who just had a three (3)-minute seizure has no apparent injuries and is oriented to name, place, and time but is very lethargic and just wants to sleep. Which intervention should the nurse implement? 1. Perform a complete neurological assessment. 2. Awaken the client every 30 minutes. 3. Turn the client to the side and allow the client to sleep. 4. Interview the client to find out what caused the seizure.

3. During the postictal (after-seizure) phase, the client is very tired and should be allowed to rest quietly; placing the client on the side will help prevent aspiration and maintain a patent airway. Reasons for other answers: 1. The client is exhausted from the seizure and should be allowed to sleep. 2. Awakening the client every 30 minutes possibly could induce another seizure as a result of sleep deprivation. 4. The client must rest, and asking questions about the seizure will keep the client awake and may induce another seizure as a result of sleep deprivation. Test-taking Hint: Options "1," "2," and "4" all have something to do with keeping the client awake. This might lead the test taker to choose the option that is different from the other three.

Which is a common cognitive problem associated with Parkinson's disease? 1. Emotional lability. 2. Depression. 3. Memory deficits. 4. Paranoia.

3. Memory deficits are cognitive impairments. The client may also develop a dementia. Test-taking Hint: The test taker must know the definitions of common medical terms. "Cognitive" refers to mental capacity to function.

The nurse is working with clients and their families regarding substance abuse. Which statement is the scientific rationale for teaching the children new coping mechanisms? 1. The child needs to realize that the parent will be changing behaviors. 2. The child will need to point out to the parent when the parent is not coping. 3. Children tend to mimic behaviors of parents when faced with similar situations. 4. Children need to feel like they are a part of the parent's recovery.

3. Most coping behaviors are learned from parents and guardians. Children of substance abusers tend to cope with life situations by becoming substance abusers unless taught healthy coping mechanisms. Test-taking Hint: Most parents do not like to be corrected by their child; this could eliminate option "2." The correct answer must address a reason for teaching new coping strategies.

The nurse has written a care plan for a client diagnosed with a brain tumor. Which is an important goal regarding self-care deficit? 1. The client will maintain body weight within two (2) pounds. 2. The client will execute an advance directive. 3. The client will be able to perform three (3) ADLs with assistance. 4. The client will verbalize feeling of loss by the end of the shift.

3. Performing activities of daily living is a goal for self-care deficit. Test-taking Hint: The test taker should read the stem of the question carefully. All of the goals could be appropriate for a client diagnosed with a brain tumor, but only one applies to self-care deficit.

The client diagnosed with Parkinson's disease is being discharged. Which statement made by the significant other indicates an understanding of the discharge instructions? 1. "All of my spouse's emotions will slow down now just like his body movements." 2. "My spouse may experience hallucinations until the medication starts working." 3. "I will schedule appointments late in the morning after his morning bath." 4. "It is fine if we don't follow a strict medication schedule on weekends."

3. Scheduling appointments late in the morning gives the client a chance to complete ADLs without pressure and allows the medications time to give the best benefits. Test-taking Hint: The test taker could eliminate option "2" because hallucinations are never an expected part of legal medication administration.

The wife of the client diagnosed with septic meningitis asks the nurse, "I am so scared. What is meningitis?" Which statement would be the most appropriate response by the nurse? 1. "There is bleeding into his brain causing irritation of the meninges." 2. "A virus has infected the brain and meninges, causing inflammation." 3. "This is a bacterial infection of the tissues that cover the brain and spinal cord." 4. "This is an inflammation of the brain parenchyma caused by a mosquito bite."

3. Septic meningitis refers to meningitis caused by bacteria; the most common form of bacterial meningitis is caused by the Neisseria meningitides bacteria. Test-taking Hint: The nurse should explain the client's diagnosis in layperson's terms when the stem is identifying the significant other as asking the question. Be sure to notice that the adjective "septic" is the key to answering this question, ruling out options "1" and "2."

Which statement by the female client indicates that the client understands factors that may precipitate seizure activity? 1. "It is all right for me to drink coffee for breakfast." 2. "My menstrual cycle will not affect my seizure disorder." 3. "I am going to take a class in stress management." 4. "I should wear dark glasses when I am out in the sun."

3. Tension states, such as anxiety and frustration, induce seizures in some clients, so stress management may be helpful in preventing seizures. Test-taking Hint: Caffeine is a stimulant and its use is not recommended in many disease processes. Menstrual cycle changes are known to affect seizure disorders. Therefore, options "1" and "2" can be eliminated, as can option "4" because sunlight does not cause seizures.

The nurse is caring for the following clients. Which client would the nurse assess first after receiving the shift report? 1. The 22-year-old male client diagnosed with a concussion who is complaining someone is waking him up every two (2) hours. 2. The 36-year-old female client admitted with complaints of left-sided weakness who is scheduled for a magnetic resonance imaging (MRI) scan. 3. The 45-year-old client admitted with blunt trauma to the head after a motorcycle accident who has a Glasgow Coma Scale score of 6. 4. The 62-year-old client diagnosed with a cerebrovascular accident (CVA) who has expressive aphasia.

3. The Glasgow Coma Scale is used to determine a client's response to stimuli (eye-opening response, best verbal response, and best motor response) secondary to a neurological problem; scores range from 3 (deep coma) to 15 (intact neurological function). A client with a score of 6 should be assessed first by the nurse. Reasons for other answers: 1. A client with a head injury must be awakened every two (2) hours to determine alertness; decreasing level of consciousness is the first indicator of increased intracranial pressure. 2. A diagnostic test, MRI, would be an expected test for a client with left-sided weakness and would not require immediate attention. 4. The nurse would expect a client diagnosed with a CVA (stroke) to have some sequelae of the problem, including the inability to speak. Test-taking Hint: This is a prioritizing question that asks the test taker to determine which client has priority when assessing all four clients. The nurse should assess the client who has abnormal data for the disease process.

Which rationale explains the transmission of the West Nile virus? 1. Transmission occurs through exchange of body fluids when sneezing and coughing. 2. Transmission occurs only through mosquito bites and not between humans. 3. Transmission can occur from human to human in blood products and breast milk. 4. Transmission occurs with direct contact from the maculopapular rash drainage.

3. The West Nile virus can be transmit- ted through breast milk, blood products, and organ transplants. This is a vector-borne disease. It is transmitted to mosquitoes that bite infected birds. The incubation period is around 15 days. Test-taking Hint: The test taker should eliminate option "2" because of the absolute word "only."

Which nursing task would be most appropriate for the nurse to delegate to the unlicensed assistive personnel? 1. Teach Credé's maneuver to the client needing to void. 2. Administer the tube feeding to the client who is quadriplegic. 3. Assist with bowel training by placing the client on the bedside commode. 4. Observe the client demonstrating self-catheterization technique.

3. The assistant can place the client on the bedside commode as part of bowel training; the nurse is responsible for the training but can delegate this task. Reasons for other answers: 1. The nurse cannot delegate assessment or teaching. 2. Tube feedings should be treated as if they were medications, and this task cannot be delegated. 4. Evaluating the client's ability to self- catheterize must be done by the nurse. Test-taking Hint: Although each state has its own delegation rules, teaching, assessing, evaluating, and medication administration are nursing interventions that cannot be delegated to unlicensed assistive personnel.

The client diagnosed with breast cancer has developed metastasis to the brain. Which prophylactic measure should the nurse implement? 1. Institute aspiration precautions. 2. Refer the client to Reach to Recovery. 3. Initiate seizure precautions. 4. Teach the client about mastectomy care.

3. The client diagnosed with metastatic lesions to the brain is at high risk for seizures. Test-taking Hint: The test taker should not read into a question—for example, about mastectomy care. The test taker should be careful to read the descriptive words in the stem; in this case "prophylactic" is the key to answering the question correctly.

The client is withdrawing from a heroin addiction. Which interventions should the nurse implement? Select all that apply. 1. Initiate seizure precautions. 2. Check vital signs every eight (8) hours. 3. Place the client in a quiet, calm atmosphere. 4. Have a consent form signed for HIV testing. 5. Provide the client with sterile needles.

3. The client should be in an atmosphere where there is little stimulation. The client will be irritable and fearful. 4. Heroin is administered intravenously. Heroin addicts are at high risk for HIV as a result of shared needles and thus should be tested for HIV. Test-taking Hint: A "select all that apply" question will usually have more than one correct answer. One option cannot eliminate another.

The client has sustained a severe closed head injury and the neurosurgeon is deter- mining if the client is "brain dead." Which data support that the client is brain dead? 1. When the client's head is turned to the right, the eyes turn to the right. 2. The electroencephalogram (EEG) has identifiable waveforms. 3. There is no eye activity when the cold caloric test is performed. 4. The client assumes decorticate posturing when painful stimuli are applied.

3. The cold caloric test, also called the oculovestibular test, is a test used to determine if the brain is intact or dead. No eye activity indicates brain death. If the client's eyes moved, that would indicate that the brainstem is intact. Reasons for other answers: 1. This is an oculocephalic test (doll's eye movement) that determines brain activity. If the eyes move with the head, it means the brainstem is intact and there is no brain death. 2. Waveforms on the EEG indicate that there is brain activity. 4. Decorticate posturing after painful stimuli are applied indicates that the brainstem is intact; flaccid paralysis is the worse neuro- logical response when assessing a client with a head injury. Test-taking Hint: The test taker needs to know what the results of the cold caloric test signify—in this case, no eye activity indicates brain death.

The nurse observes a coworker acting erratically. The clients assigned to this coworker don't seem to get relief when pain medications are administered. Which action should the nurse take? 1. Try to help the coworker by confronting the coworker with the nurse's suspicions. 2. Tell the coworker that the nurse will give all narcotic medications from now on. 3. Report the nurse's suspicions to the nurse's supervisor or the facility's peer review. 4. Do nothing until the nurse can prove the coworker has been using drugs.

3. The coworker's supervisor or peer review committee should be aware of the nurse's suspicions so that the suspicions can be investigated. This is a client safety and care concern. Test-taking Hint: The test taker can eliminate option "4" on the basis of "do nothing." In this instance, direct confrontation is not recommended, but the nurse must do something—namely, report the suspicions to the supervisor or peer review.

The client is scheduled for an electroencephalogram (EEG) to help diagnose a seizure disorder. Which teaching should the nurse implement? 1. Tell the client to take any routine antiseizure medication prior to the EEG. 2. Tell the client not to eat anything for eight (8) hours prior to the procedure. 3. Instruct the client to stay awake for 24 hours prior to the EEG. 4. Explain to the client that there will be some discomfort during the procedure.

3. The goal is for the client to have a seizure during the EEG. Sleep deprivation, hyperventilating, or flashing lights may induce a seizure. Reasons for other answers: 1. Antiseizure drugs, tranquilizers, stimulants, and depressants are withheld before an EEG because they may alter the brain wave patterns. 2. Meals are not withheld because altered blood glucose level can cause changes in brain wave patterns. 4. Electrodes are placed on the client's scalp, but there are no electroshocks or any type of discomfort. Test-taking Hint: The test taker should highlight the words "diagnose a seizure disorder" in the stem and ask which answer option would possibly cause a seizure.

The nurse arrives at the site of a one-car motor-vehicle accident and stops to render aid. The driver of the car is unconscious. After stabilizing the client's cervical spine, which action should the nurse take next? 1. Carefully remove the driver from the car. 2. Assess the client's pupils for reaction. 3. Assess the client's airway. 4. Attempt to wake the client up by shaking him.

3. The nurse must maintain a patent airway. Airway is the first step in resuscitation. Reasons for other answers: 1. The nurse should stabilize the client's neck prior to removal from the car. 2. The nurse must stabilize the client's neck before doing any further assessment. Most nurses don't carry penlights, and the client's pupil reaction can be determined after stabilization. 4. Shaking the patient could cause further damage, possibly leading to paralysis. Test-taking Hint: Remember that, in a question asking about which action should be taken first, all of the answers are interventions, but only one should be implemented first. There are very few "always" situations in the health-care profession, but in this situation, unless the client's car is on fire or under water, stabilizing the client's neck is always priority, followed by airway.

The client admitted to the hospital to rule out encephalitis is being prepared for a lumbar puncture. Which instructions should the nurse teach the client regarding care postprocedure? 1. Instruct that all invasive procedures require a written permission. 2. Explain that this allows analysis of a sample of the cerebrospinal fluid. 3. Tell the client to increase fluid intake to 300 mL for the next 48 hours. 4. Discuss that lying supine with the head flat will prevent all hematomas.

3. The nurse should teach this information to prevent the severe, throbbing, "spinal headache" caused by the decrease in cerebrospinal fluid. Test-taking Hint: When the test taker is trying to eliminate options, any that have absolute words, such as "all," "never," and "always," are usually wrong and can be eliminated quickly. Rarely is any activity always or never done.

The client with a closed head injury has clear fluid draining from the nose. Which action should the nurse implement first? 1. Notify the health-care provider immediately. 2. Prepare to administer an antihistamine. 3. Test the drainage for presence of glucose. 4. Place 2 × 2 gauze under the nose to collect drainage.

3. The presence of glucose in drainage from the nose or ears indicates cerebrospinal fluid, and the HCP should be notified immediately once this is determined. Reasons for other answers: 1. Prior to notifying the HCP, the nurse should always make sure that all the needed assessment information is available to discuss with the HCP. 2. With head injuries, any clear drainage may indicate a cerebrospinal fluid leak; the nurse should not assume the drainage is secondary to allergies and administer an antihistamine. 4. This would be appropriate, but it is not the first intervention. The nurse must determine where the fluid is coming from. Test-taking Hint: The question is asking which intervention should be implemented first, and the nurse should always assess the situation before calling the HCP or taking an action.

The son of a client diagnosed with ALS asks the nurse, "Is there any chance that I could get this disease?" Which statement by the nurse would be most appropriate? 1. "It must be scary to think you might get this disease." 2. "No, this disease is not genetic or contagious." 3. "ALS does have a genetic factor and runs in families." 4. "If you are exposed to the same virus, you may get the disease."

3. There is a genetic factor with ALS that is linked to a chromosome 21 defect. Test-taking Hint: This question re- quires knowledge of ALS. There are some questions for which Test-Taking Hints are not available.

The nurse and an unlicensed assistive personnel (UAP) are caring for a client with right-sided paralysis. Which action by the UAP requires the nurse to intervene? 1. The assistant places a gait belt around the client's waist prior to ambulating. 2. The assistant places the client on the back with the client's head to the side. 3. The assistant places a hand under the client's right axilla to move up in bed. 4. The assistant praises the client for attempting to perform ADLs independently.

3. This action is inappropriate and would require intervention by the nurse because pulling on a flaccid shoulder joint could cause shoulder dislocation; the client should be pulled up by placing the arm underneath the back or using a lift sheet. Reasons for other answers: 1. Placing a gait belt prior to ambulating is an appropriate action for safety and would not require the nurse to intervene. 2. Placing the client in a supine position with the head turned to the side is not a problem position, so the nurse does not need to intervene. 4. The client should be encouraged and praised for attempting to perform any activities independently, such as combing hair or brushing teeth. Test-taking Hint: This type of question has three answer options that do not require a nurse to intervene to correct a subordinate. Remember to read every possible answer option before deciding on a correct one.

The client with a C6 SCI is admitted to the emergency department complaining of a severe pounding headache and has a BP of 180/110. Which intervention should the emergency department nurse implement? 1. Keep the client flat in bed. 2. Dim the lights in the room. 3. Assess for bladder distention. 4. Administer a narcotic analgesic.

3. This is an acute emergency caused by exaggerated autonomic responses to stimuli and only occurs after spinal shock has resolved in the client with a spinal cord injury above T6. The most common cause is a full bladder. Reasons for other answers: 1. This action will not address the client's pounding headache and hypertension. 2. Dimming the lights will not help the client's condition. 4. The nurse should always assess the client before administering medication. Test-taking Hint: The test taker should apply the nursing process when answering questions, and assessing the client comes first, before administering any type of medication.

Which assessment data would indicate to the nurse that the client would be at risk for a hemorrhagic stroke? 1. A blood glucose level of 480 mg/dL. 2. A right-sided carotid bruit. 3. A blood pressure of 220/120 mm Hg. 4. The presence of bronchogenic carcinoma.

3. Uncontrolled hypertension is a risk factor for hemorrhagic stroke, which is a ruptured blood vessel inside the cranium. Reasons for other answers: 1. This glucose level is elevated and could predispose the client to ischemic neurological changes due to blood viscosity, but it is not a risk factor for a hemorrhagic stroke. 2. A carotid bruit predisposes the client to an embolic or ischemic stroke but not to a hemorrhagic stroke. 4. Cancer is not a precursor to developing a hemorrhagic stroke. Test-taking Hint: Both options "1" and "2" are risk factors for an ischemic or embolic type of stroke. Knowing this, the test taker can rule out these options as incorrect.

The nurse is planning care for a client experiencing agnosia secondary to a cerebrovas- cular accident. Which collaborative intervention will be included in the plan of care? 1. Observe the client swallowing for possible aspiration. 2. Position the client in a semi-Fowler's position when sleeping. 3. Place a suction setup at the client's bedside during meals. 4. Refer the client to an occupational therapist for evaluation.

4. A collaborative intervention is an intervention in which another health-care discipline—in this case, occupational therapy—is used in the care of the client. Reasons for other answers: 1. Agnosia is the failure to recognize familiar objects; therefore, observing the client for possible aspiration is not appropriate. 2. A semi-Fowler's position is appropriate for sleeping, but agnosia is the failure to recognize familiar objects; therefore, this intervention is inappropriate. 3. Placing suction at the bedside will help if the client has dysphagia (difficulty swallowing), not agnosia, which is failure to recognize familiar objects. Test-taking Hint: Be sure to look at what the question is asking and see if the answer can be determined even if some terms are not understood. In this case, note that the question refers to "collaborative interven- tion." Only option "4" refers to collaboration with another discipline.

The client is being evaluated to rule out ALS. Which signs/symptoms would the nurse note to confirm the diagnosis? 1. Muscle atrophy and flaccidity. 2. Fatigue and malnutrition. 3. Slurred speech and dysphagia. 4. Weakness and paralysis.

4. ALS results from the degeneration and demyelination of motor neurons in the spinal cord, which results in paralysis and weakness of the muscles. Test-taking Hint: This is an application question in which the test taker must know that ruling out of ALS would re- sult in the answer being early signs/symptoms. The test taker could rule out option "1" because of atrophy, which is a long-term occurrence; rule out option "2" because these symptoms will occur as the disease progresses; and rule out option "3" because these are late signs/symptoms.

Which collaborative intervention should the nurse implement when caring for the client with West Nile virus? 1. Complete neurovascular examinations every eight (8) hours. 2. Maintain accurate intake and output at the end of each shift. 3. Assess the client's symptoms to determine if there is improvement. 4. Administer intravenous fluids while assessing for overload.

4. Administering an IV fluid is collabo- rative because it requires an order from a health-care provider. It does, however, require the nurse to assess the rate, fluid, and site for complications. Test-taking Hint: When reading test questions, the test taker should pay attention to adjectives. In this question, the word "collaborative" makes all the options incorrect except option "4." Collaborative interventions require an order from a health-care provider, but the nurse uses judgment and intuition within the scope of practice.

The friend of an 18-year-old male client brings the client to the emergency department (ED). The client is unconscious and his breathing is slow and shallow. Which action should the nurse implement first? 1. Ask the friend what drugs the client has been taking. 2. Initiate an IV infusion at a keep-open rate. 3. Call for a ventilator to be brought to the ED. 4. Apply oxygen at 100% via nasal cannula.

4. Applying oxygen would be the priority action for this client. The client's breathing is slow and shallow. The greater amount of inhaled oxygen, the better the client's prognosis. Test-taking Hint: When the test taker is deciding on a priority, some guidelines should be used. Maslow's hierarchy of needs places oxygen at the top of the priority list. In cardiopulmonary resuscitation, airway and breathing are first.

The client is diagnosed with Wernicke-Korsakoff syndrome as a result of chronic alcoholism. For which symptoms would the nurse assess? 1. Insomnia and anxiety. 2. Visual or auditory hallucinations. 3. Extreme tremors and agitation. 4. Ataxia and confabulation.

4. Ataxia, or lack of coordination, and confabulation, making up elaborate stories to explain lapses in memory, are both symptoms of Wernicke-Korsakoff syndrome. Test-taking Hint: The test taker can eliminate options "2" and "3" if the test taker knows the symptoms of delirium tremens.

Which type of precautions should the nurse implement for the client diagnosed with septic meningitis? 1. Standard Precautions. 2. Airborne Precautions. 3. Contact Precautions. 4. Droplet Precautions.

4. Droplet Precautions are respiratory precautions used for organisms that have a limited span of transmission. Precautions include staying at least four (4) feet away from the client or wearing a standard isolation mask and gloves when coming in close contact with the client. Clients are in isolation for 24 to 48 hours after initiation of antibiotics. Test-taking Hint: The test taker must know the types of isolation precautions used for different diseases and note the adjective—"septic"—in the stem of the question.

The nurse researcher is working with clients diagnosed with Parkinson's disease. Which is an example of an experimental therapy? 1. Sterotactic pallidotomy/thalamotomy. 2. Dopamine receptor agonist medication. 3. Physical therapy for muscle strengthening. 4. Fetal tissue transplantation.

4. Fetal tissue transplantation has shown some success in PD, but it is an experimental and highly controversial procedure. Test-taking Hint: The test taker should not overlook the adjective "experimental." This would eliminate at least option "3," physical therapy, and option "2," which refers to standard dopamine treatment, even if the test taker was not familiar with all of the procedures.

The intensive care nurse is caring for a client with a T1 SCI. When the nurse elevates the head of the bed 30 degrees, the client complains of lightheadedness and dizziness. The client's vital signs are T 99.2 ̊F, P 98, R 24, and BP 84/40. Which action should the nurse implement? 1. Notify the health-care provider ASAP. 2. Calm the client down by talking therapeutically. 3. Increase the IV rate by 50 mL/hour. 4. Lower the head of the bed immediately.

4. For the first two (2) weeks after an SCI above T7, the blood pressure tends to be unstable and low; slight elevations of the head of the bed can cause profound hypotension; therefore, the nurse should lower the head of the bed immediately. Reasons for other answers: 1. This is not an emergency; therefore, the nurse should not notify the health-care provider. 2. A physiological change in the client requires more than a therapeutic conversation. 3. Increasing the IV rate will not address the cause of the problem. Test-taking Hint: The test taker should notice that the only answer option that addresses the "bed" is the correct answer. This does not always help identify the correct answer, but it is a hint that should be used if the test taker has no idea what the correct answer is.

The client has been diagnosed with a brain tumor. Which presenting signs and symptoms help to localize the tumor position? 1. Widening pulse pressure and bounding pulse. 2. Diplopia and decreased visual acuity. 3. Bradykinesia and scanning speech. 4. Hemiparesis and personality changes.

4. Hemiparesis would localize a tumor to a motor area of the brain, and personality changes localize a tumor to the frontal lobe. Test-taking Hint: The test taker could arrive at the correct answer if the test taker realized that specific regions of the brain control motor function and hemiparesis and that other regions are involved in personality changes.

The client is diagnosed with an SCI and is scheduled for a magnetic resonance imaging (MRI) scan. Which question would be most appropriate for the nurse to ask prior to taking the client to the diagnostic test? 1. "Do you have trouble hearing?" 2. "Are you allergic to any type of dairy products?" 3. "Have you eaten anything in the last eight (8) hours?" 4. "Are you uncomfortable in closed spaces?"

4. MRI scans are often done in a very confined space; many people who have claustrophobia must be medicated or even rescheduled for the procedure in an open MRI machine, which may be available if needed. Reasons for other answers: 1. The machine is very loud and the technician will offer the client ear plugs, but hearing difficulty will not affect the MRI scan. 2. Allergies to dairy products will not affect the MRI scan. 3. The client does not need to be NPO for this procedure. Test-taking Hint: The nurse must be knowledgeable of diagnostic tests to prepare the client for the tests safely. The test taker must be realistic in determining answers—is there any test in which a hearing problem would make the diagnostic test contraindicated?

The nurse is admitting a client with the diagnosis of Parkinson's disease. Which assessment data support this diagnosis? 1. Crackles in the upper lung fields and jugular vein distention. 2. Muscle weakness in the upper extremities and ptosis. 3. Exaggerated arm swinging and scanning speech. 4. Masklike facies and a shuffling gait.

4. Masklike facies and a shuffling gait are two clinical manifestations of PD. Test-taking Hint: Option "3" refers to arm swing and speech, both of which are affected by PD. The test taker needs to decide if the adjectives used to describe these activities—"exaggerated" and "scanning"—are appropriate. They are not, but masklike facies and shuffling gait are.

The 29-year-old client is admitted to the medical floor diagnosed with meningitis. Which assessment by the nurse has priority? 1. Assess lung sounds. 2. Assess the six cardinal fields of gaze. 3. Assess apical pulse. 4. Assess level of consciousness.

4. Meningitis directly affects the client's brain. Therefore, assessing the neurological status would have priority for this client. Test-taking Hint: The test taker should apply a systemic approach to discerning the priority response. Maslow's hierarchy of needs would put option "1" as correct, but the disease process of meningitis does not include signs or symptoms of a respiratory component. The next highest priority would be the neurological component, and meningitis definitely is a neurological disease.

The client has been diagnosed with a cerebrovascular accident (stroke). The client's wife is concerned about her husband's generalized weakness. Which home modification should the nurse suggest to the wife prior to discharge? 1. Obtain a rubber mat to place under the dinner plate. 2. Purchase a long-handled bath sponge for showering. 3. Purchase clothes with Velcro closure devices. 4. Obtain a raised toilet seat for the client's bathroom.

4. Raising the toilet seat is modifying the home and addresses the client's weakness in being able to sit down and get up without straining muscles or requiring lifting assistance from the wife. Reasons for other answers: 1. The rubber mat will stabilize the plate and prevent it from slipping away from the client learning to feed himself, but this does not address generalized weakness. 2. A long-handled bath sponge will assist the client when showering hard-to-reach areas, but it is not a home modification, nor will it help with generalized weakness. 3. Clothes with Velcro closures will make dressing easier, but they do not constitute a home modification and do not address generalized weakness. Test-taking Hint: The test taker must read the stem of the question carefully and note that the intervention must be one in which the home is modified in some way. This would eliminate three of the options, leaving the correct answer.

The nurse is assessing the client admitted with encephalitis. Which data require immediate nursing intervention? The client: 1. Has bilateral facial palsies. 2. Has a recurrent temperature of 100.6 ̊F. 3. Has a decreased complaint of headache. 4. Comments that the meal has no taste.

4. The absence of smell and taste indicates that the cranial nerves may be involved. The client's condition is becoming more serious. Test-taking Hint: This question requires the test taker to select an option that indicates the disease is progressing and the client is at risk. Option "3" indicates that the client is improving, and options "1" and "2" are common early manifestations of the disease. The only option that reflects cranial nerve involve- ment, a sign that the client's condition is becoming worse and requires immediate intervention, is option "4."

The male client is sitting in the chair and his entire body is rigid with his arms and legs contracting and relaxing. The client is not aware of what is going on and is making guttural sounds. Which action should the nurse implement first? 1. Push aside any furniture. 2. Place the client on his side. 3. Assess the client's vital signs. 4. Ease the client to the floor.

4. The client should not remain in the chair during a seizure. He should be brought safely to the floor so that he will have room to move the extremities. Reasons for other answers: 1. The nurse needs to protect the client from injury. Moving furniture would help ensure that the client would not hit something accidentally, but this is not done first. 2. This is done to help keep the airway patent, but it is not the first intervention in this specific situation. 3. Assessment is important but, when the client is having a seizure, the nurse should not touch him or her. Test-taking Hint: All of the answer options are possible interventions, so the test taker should go back to the stem of the question and note that the question asks which intervention has priority. "In the chair" is the key to this question because the nurse should always think about safety, and a patient having a seizure is not safe in a chair.

The nurse is planning the care for a client diagnosed with Parkinson's disease. Which would be a therapeutic goal of treatment for the disease process? 1. The client will experience periods of akinesia throughout the day. 2. The client will take the prescribed medications correctly. 3. The client will be able to enjoy a family outing with the spouse. 4. The client will be able to carry out activities of daily living.

4. The major goal of treating PD is to maintain the ability to function. Clients diagnosed with PD experience slow, jerky movements and have difficulty performing routine daily tasks. Test-taking Hint: The test taker should match the goal to the problem. A "therapeutic goal" is the key to answering this question.

The client diagnosed with ALS is prescribed an antiglutamate, riluzole (Rilutek). Which instruction should the nurse discuss with the client? 1. Take the medication with food. 2. Do not eat green, leafy vegetables. 3. Use SPF 30 when going out in the sun. 4. Report any febrile illness.

4. The medication can cause blood dyscrasias. Therefore, the client is monitored for liver function, blood count, blood chemistries, and alkaline phosphatase. The client should report any febrile illness. This is the first medication developed to treat ALS. Test-taking Hint: Blood dyscrasias occur with many medications, and this might prompt the test taker to select option "4" as the correct option. Other- wise, the test taker must be knowledge- able of medication administration.

The client is admitted to the medical floor with a diagnosis of closed head injury. Which nursing intervention has priority? 1. Assess neurological status. 2. Monitor pulse, respiration, and blood pressure. 3. Initiate an intravenous access. 4. Maintain an adequate airway.

4. The most important nursing goal in the management of a client with a head injury is to establish and maintain an adequate airway. Reasons for other answers: 1. Assessing the neurological status is important, but ensuring an airway is priority over assessment. 2. Monitoring vital signs is important, but maintaining an adequate airway is higher priority. 3. Initiating an IV access is an intervention the nurse can implement, but it is not the priority intervention. Test-taking Hint: If the question asks for a priority intervention, it means that all of the options would be appropriate for the client but only one intervention is priority. Always apply Maslow's hierarchy of needs—an adequate airway is first.

The client diagnosed with a brain tumor was admitted to the intensive care unit with decorticate posturing. Which indicates that the client's condition is becoming worse? 1. The client has purposeful movement with painful stimuli. 2. The client has assumed adduction of the upper extremities. 3. The client is aimlessly thrashing in the bed. 4. The client has become flaccid and does not respond to stimuli.

4. The most severe neurological impairment result is flaccidity and no response to stimuli. This indicates that the client's condition has worsened. Test-taking Hint: Neurological assessment includes assessing the client for levels of consciousness; the nurse must memorize the stages of neurological progression toward a coma and death.

The 85-year-old client diagnosed with a stroke is complaining of a severe headache. Which intervention should the nurse implement first? 1. Administer a nonnarcotic analgesic. 2. Prepare for STAT magnetic resonance imaging (MRI). 3. Start an intravenous infusion with D5W at 100 mL/hr. 4. Complete a neurological assessment.

4. The nurse must complete a neurological assessment to help determine the cause of the headache before taking any further action. Reasons for other answers: 1. The nurse should not administer any medication to a client without first assessing the cause of the client's complaint or problem. 2. An MRI scan may be needed, but the nurse must determine the client's neurological status prior to diagnostic tests. 3. Starting an IV infusion is appropriate, but it is not the action the nurse should implement when assessing pain, and 100 mL/hr might be too high a rate for an 85-year-old client. Test-taking Hint: The test taker should always apply the nursing process when answering questions. If the test taker narrows down the choices to two possible answer options, always select the assessment option as the first intervention.

The nurse is enjoying a day at the lake and witnesses a water skier hit the boat ramp. The water skier is in the water not responding to verbal stimuli. The nurse is the first health-care provider to respond to the accident. Which intervention should be implemented first? 1. Assess the client's level of consciousness. 2. Organize onlookers to remove the client from the lake. 3. Perform a head-to-toe assessment to determine injuries. 4. Stabilize the client's cervical spine.

4. The nurse should always assume that a client with traumatic head injury may have sustained spinal cord injury. Mov- ing the client could further injure the spinal cord and cause paralysis; there- fore, the nurse should stabilize the cervical spinal cord as best as possible prior to removing the client from the water. Reasons for other answers: 1. Assessment is important, but with clients with head injury the nurse must assume spinal cord injury until it is ruled out with x-ray; therefore, stabilizing the spinal cord is priority. 2. Removing the client from the water is an appropriate intervention, but the nurse must assume spinal cord injury until it is ruled out with x-ray; therefore, stabilizing the spinal cord is priority. 3. Assessing the client for further injury is appropriate, but the first intervention is to stabilize the spine because the impact was strong enough to render the client unconsciousness. Test-taking Hint: When two possible answer options contain the same directive word—in this case, "assess"—the test taker can either rule out these two as incorrect or prioritize between the two assessment responses.

The wife of the client diagnosed with chronic alcoholism tells the nurse, "I have to call his work just about every Monday to let them know he is ill or he will lose his job." Which would be the nurse's best response? 1. "I am sure that this must be hard for you. Tell me about your concerns." 2. "You are afraid he will lose his source of income." 3. "Why would you call in for your husband? Can't he do this?" 4. "Are you aware that when you do this you are enabling him?"

4. The spouse's behavior is enabling the client to continue to drink until he cannot function. Test-taking Hint: The stem of the question did not ask for a therapeutic response but did ask for the nurse's best response. The best response is to address the problem.

The nurse is caring for the client diagnosed with West Nile virus. Which assessment data would require immediate intervention from the nurse? 1. The vital signs are documented as T 100.2 ̊F, P 80, R 18, and BP 136/78. 2. The client complains of generalized body aches and pains. 3. Positive results from the enzyme-linked immunosorbent assay (ELISA). 4. The client becomes lethargic and is difficult to arouse using verbal stimuli.

4. These assessment data may indicate that the client's condition is deterio- rating and require immediate inter- vention to prevent complications. Test-taking Hint: The word "immedi- ate" means that the nurse must recog- nize and intervene before complications occur. The test taker should eliminate any option that contains normal assessment data.

The client diagnosed with a brain tumor has a diminished gag response. Which intervention should the nurse implement? 1. Make the client NPO until seen by the health-care provider. 2. Position the client in low Fowler's position for all meals. 3. Place the client on a mechanically ground diet. 4. Teach the client to direct food and fluid toward the unaffected side.

4. To decrease the risk of aspiration, the client should direct food to the unaffected side of the throat; this helps the client to be able to use the side of the throat that is functioning. Test-taking Hint: The test taker should try to visualize the position of the client in the bed. A mostly recumbent position (low Fowler's) would increase the chance of aspiration; thus option "2" should be eliminated.

The nurse caring for a client diagnosed with Parkinson's disease writes a problem of "impaired nutrition." Which nursing intervention would be included in the plan of care? 1. Consult the occupational therapist for adaptive appliances for eating. 2. Request a low-fat, low-sodium diet from the dietary department. 3. Provide three (3) meals per day that include nuts and whole-grain breads. 4. Offer six (6) meals per day with a soft consistency.

4.The client's energy levels will not sustain eating for long periods. Offering frequent and easy-to-chew (soft) meals of small proportions is the preferred dietary plan. Test-taking Hint: he correct answer for a nursing problem question must address the actual problem.

The nurse is caring for the following clients on a medical unit. Which client should the nurse assess first? 1. The client with ALS who is refusing to turn every two (2) hours. 2. The client with abdominal pain who is complaining of nausea. 3. The client with pneumonia who has a pulse oximeter reading of 90%. 4. The client who is complaining about not receiving any pain medication.

A pulse oximeter reading of less than 93% indicates that the client is experiencing hypoxemia, which is a life-threatening emergency. This client should be assessed first. Test-taking Hint: The test taker should apply Maslow's hierarchy of needs, in which oxygenation is priority. The nurse must know normal parameters for diagnostic tools and laboratory data.

The client diagnosed with a mild concussion is being discharged from the emergency department. Which discharge instruction should the nurse teach the client's significant other? 1. Awaken the client every two (2) hours. 2. Monitor for increased intracranial pressure. 3. Observe frequently for hypervigilance. 4. Offer the client food every three (3) to four (4) hours.

Awakening the client every two (2) hours allows the identification of headache, dizziness, lethargy, irritability, and anxiety—all signs of post-concussion syndrome—that would warrant the significant other's taking the client back to the emergency department. Reasons for other answers: 2. The nurse should monitor for signs of increased intracranial pressure (ICP), but a layman, the significant other, would not know what these signs and medical terms mean. 3. Hypervigilance, increased alertness and super-awareness of the surroundings, is a sign of amphetamine or cocaine abuse, but it would not be expected in a client with a head injury. 4. The client can eat food as tolerated, but feeding the client every three (3) to four (4) hours does not affect the development of post-concussion syndrome, the signs of which are what should be taught to the significant other. Test-taking Hint: Remember to pay close attention to answer options that have times (e.g., "every two [2] hours," "every three [3] to four [4] hours"). Also consider the likelihood of the options listed. Would a nurse teach the significant other terms such as increased intracranial pressure or hypervigilance? Probably not, so options "2" and "3" should be eliminated.

The home health nurse is caring for a 28-year-old client with a T10 SCI who says, "I can't do anything. Why am I so worthless?" Which statement by the nurse would be the most therapeutic? 1. "This must be very hard for you. You're feeling worthless?" 2. "You shouldn't feel worthless—you are still alive." 3. "Why do you feel worthless? You still have the use of your arms." 4. "If you attended a work rehab program you wouldn't feel worthless."

Therapeutic communication addresses the client's feelings and attempts to allow the client to verbalize feelings; the nurse should be a therapeutic listener. Reasons for other answers: 2. This is belittling the client's feelings. 3. The client does not owe the nurse an explanation of his feelings; "why" is never therapeutic. 4. This is advising the client and is not therapeutic. Test-taking Hint: When the question requests a therapeutic response, the test taker should select the answer option that has "feelings" in the response.


Related study sets

Machine Learning True False questions

View Set

Chapter 1: General Concepts and Administrative Issues

View Set

Microeconomics HW Questions Ch. 5-8

View Set

CPR Online Portion Study Guide 2021 January

View Set

The Radicals Take Control Chapter 2 Lesson 2

View Set